2020 AMC 12A Problems/Problem 17

Revision as of 19:30, 1 February 2020 by Giacomorizzo (talk | contribs) (Problem 17)
(diff) ← Older revision | Latest revision (diff) | Newer revision → (diff)

Problem 17

The vertices of a quadrilateral lie on the graph of $y=\ln{x}$, and the $x$-coordinates of these vertices are consecutive positive integers. The area of the quadrilateral is $\ln{\frac{91}{90}}$. What is the $x$-coordinate of the leftmost vertex?

$\textbf{(A) } 6 \qquad \textbf{(B) } 7 \qquad \textbf{(C) } 10 \qquad \textbf{(D) } 12 \qquad \textbf{(E) } 13$

Solution 1